Jump to content

Photo

Đơn giản

- - - - -

  • Please log in to reply
4 replies to this topic

#1
MyLoveIs4Ever

MyLoveIs4Ever

    Sĩ quan

  • Thành viên
  • 441 posts
CMR nếu $ 2^n \equiv -1 ( mod 3^k ) $ thì $ n | 3^{k-1} $

#2
Bé con

Bé con

    Binh nhất

  • Thành viên
  • 44 posts
Liều mạng làm thử,sai cấm cười:
Ta có:
2^{2. 3^{k-1} } đồng dư 1 (mod 3^{k})
2^{2n} đồng dư 1 (mod 3^{k})
Suy ra 2^{2 (3^{k-1},n) } đồng dư 1 (mod 3^{k})
Nếu n không là ước 3^{k-1} suy ra (n,3^{k-1})=1 suy ra 4 đồng dư 1 (mod 3^{k}) suy ra k=1 suy ra bài toán sai nếu n=3,k=1
Nếu k>1 thì từ 4 đồng dư 1 mod 3^{k} dẫn đến mâu thuẫn,chứng tỏ bài toán đúng nếu k>1
(Đừng xóa bài em,em đang học đánh Latex)

#3
H.Quân- ĐHV

H.Quân- ĐHV

    An-tôn Páp-lô-vích Sê-Khốp

  • Thành viên
  • 530 posts

CMR nếu $ 2^n \equiv -1 ( mod 3^k ) $ thì $ n | 3^{k-1} $

ta có$ (2,3^k) = 1$ nên

$ 2^{\phi (3^k) } \equiv 1 ( mod 3^k) $ :D $ 4^{3^{k-1} } \equiv 1 ( mod 3^k)$
mà $4^{n} \equiv 1 ( mod 3^k)$ nên $(n , 3^{k-1} ) = h$ ($ h$ là cấp )

thế thi $h = 3^t$ ta cm $t= k-1 $thật vậy nếu$ t<k-1$

ta có $4^{3^t} - 1 = 3 ( 4^{ 3^t - 1 } + 4^{3^t - 2} + ...+ 4+ 1 ) \vdots 3^k $

mà $4^{3^t - i} $ với $i=1 ,2,...,3^t -1 $ lập thành 1 hệ thặng dư đầy đủ mod $3^{k-1}$ ( do $ t $ min )

khi đó $4^{ 3^t - 1 } + 4^{3^t - 2} + ...+ 4+ 1 \equiv 3^t(3^t+1) : 2 ( mod 3^{k-1} ) $ :Rightarrow$ t = k-1 $ đpcm

Edited by H.Quân- ĐHV, 16-04-2008 - 22:08.

I hope for the best

Chẳng có gì đáng giá bằng nụ cười và tình yêu thương của bạn bè

Trên bước đường thành công không có dấu chân của kẻ lười biếng

#4
T*genie*

T*genie*

    Đường xa nặng bóng ngựa lười...

  • Quản lý Toán Ứng dụng
  • 1161 posts

(Đừng xóa bài em,em đang học đánh Latex)


Anh gõ lại cho em được chứ?

$2^{2. 3^{k-1} }$ đồng dư 1 (mod $3^{k}$)
$2^{2n}$ đồng dư 1 (mod $3^{k}$)
Suy ra $2^{2 (3^{k-1},n) }$ đồng dư 1 (mod $3^{k}$)
Nếu n không là ước $3^{k-1}$ suy ra (n,$3^{k-1}$)=1 suy ra 4 đồng dư 1 (mod $3^{k}$) suy ra k=1 suy ra bài toán sai nếu n=3,k=1
Nếu k>1 thì từ 4 đồng dư 1 mod $3^{k}$ dẫn đến mâu thuẫn,chứng tỏ bài toán đúng nếu k>1


em chỉ cần chèn tex khi đánh công thức là ok.

#5
MyLoveIs4Ever

MyLoveIs4Ever

    Sĩ quan

  • Thành viên
  • 441 posts
Bài nì nhiều cách CM lém , tạm thời có 2 cách nữa ( ý tưởng vẫn là căn nguyên thủy nhưng đi theo 2 hướng)

Dùng định lí sau
Với $ p $ nguyên tố lẻ nếu $ r $ là căn nguyên thủy modulo $ p $ và $ r $ cũng là căn nguyên thủy modulo $ p^2 $ thì $ r $ cũng là căn nguyên thủy modulo $ p^k $ mọi $ k \geq 3 $

CM ko khó mấy

Áp dụng vào là ok

Hoặc xét riêng bài toán này thì Chỉ cần CM $ 2 $ là căn nguyên thủy modulo $ 3^n $ là đủ
hay dùng bổ đề sau:
$ 2^{2.3^{n-1}} \equiv 1+3^n (mod 3^{n+1}) $




1 user(s) are reading this topic

0 members, 1 guests, 0 anonymous users